Đến nội dung

nhungvienkimcuong nội dung

Có 209 mục bởi nhungvienkimcuong (Tìm giới hạn từ 10-06-2020)



Sắp theo                Sắp xếp  

#729193 CMR: $a_n\equiv 2,3,6\pmod{8}$

Đã gửi bởi nhungvienkimcuong on 28-07-2021 - 10:53 trong Số học

Tham khảo ở đây. Kết quả bài này  là $a_n\equiv 2,3\pmod{8}$.

Một bài tương tự là

$$\left \lfloor \left ( \sqrt[3]{65}-4 \right )^{-n} \right \rfloor\equiv 2,3\pmod{15}$$

 




#731306 [TOPIC] Mỗi ngày một bài toán IMO

Đã gửi bởi nhungvienkimcuong on 25-10-2021 - 15:29 trong Các dạng toán khác

Bài 8: [IMO 1987] Chứng minh rằng không tồn tại hàm $f$ nào từ tập hợp các số nguyên không âm vào chính nó thoả mãn $f(f(n))=n+1987$ với mọi $n$.

 

Bài này ta có thể tổng quát lên như sau: Tồn tại hàm số $f\colon \mathbb{N}\to \mathbb{N}$ sao cho $\underbrace{f(f(\cdots f}_{a\ \text{lần}}(n)\cdots))=n+b$ với mọi $n\in \mathbb{N}$ khi và chỉ khi $a\mid b$

 

Ở đây mình chỉ làm theo đề gốc, bài toán tổng quát các bạn dành cho các bạn  :D

Giả sử tồn tại hàm $f$ thỏa đề. Từ giả thiết dễ thấy $f(n+1987)=f(n)+1987\ \color{Red}{(1)}$.

Tiếp đến ta làm việc trong $ \mathbb{Z}_{1987}$, có được

$$f(f(x))=x,\quad \forall x\in \mathbb{Z}_{1987}$$

Xây dựng đồ thị có hướng $[V,E]$ trong đó tập đỉnh chính là các phần tử của tập hợp $\mathbb{Z}_{1987}$, có cạnh $[v_1,v_2]$ nếu $v_2=f(v_1)$. Với cách xây dựng này thì ta thấy mỗi đỉnh của đồ thị thuộc $1$-chu trình hoặc $2$-chu trình. Vì $1987$ là số lẻ nên tồn tại điểm bất điểm bất động $m\in \mathbb{Z}_{1987}$, nghĩa là $f(m)=m$.

Quay trở lại làm việc trong $\mathbb{Z}$. ta có được $f(m)=m+1987k$ với $k$ là số nguyên nào đó. Do vậy

\[f(f(m))=f(m+1987k)\overset{\color{Red}{(1)}}{=}f(m)+1987k=m+1987\cdot 2k\]

Mặt khác theo giả thiết thì $f(f(m))=m+1987$, suy ra $2k=1$ (vô lí).




#731456 $$\exists_{n}^{\infty}\;n^{...

Đã gửi bởi nhungvienkimcuong on 03-11-2021 - 14:03 trong Số học

Bạn cũng có thể thử bài này $\exists_{n}^{\infty}\;n^{2}+ 1\nmid n!\!$ HOẶC chứng minh mệnh đề của bạn với số nguyên dương $n$ thứ $k$ sao cho $n\leq 18k.$




#731580 $n\sqrt{d}\left \{ n\sqrt{d...

Đã gửi bởi nhungvienkimcuong on 11-11-2021 - 10:41 trong Số học

$\boxed{\text{Problem 2}}$ (Balkan MO 2015)

Chứng minh rằng trong $20$ số nguyên dương liên tiếp có một số nguyên dương $d$ sao cho với mọi số nguyên dương $n$, bất đẳng thức sau đúng:

$n\sqrt{d}\left \{ n\sqrt{d} \right \}>\frac{5}{2}$

Ý tưởng câu này tương tự ở đây

 

Đặt $m=\left \lfloor n\sqrt{d} \right \rfloor$. Khi đó

$$n^2d-m^2=\left \{ n\sqrt{d} \right \}(n\sqrt{d}+m)\le 2\left \{ n\sqrt{d} \right \}n\sqrt{d}$$

Ta sẽ chọn $d$ không phải là số chính phương $(1)$, dẫn tới

\[n\sqrt{d}\left \{ n\sqrt{d} \right \}>\frac{n^2d-m^2}{2}\]

Tiếp đến ta sẽ chọn $d$ phù hợp sao cho $n^2d-m^2\notin \{1,2,3,4\}$ $(2)$

 

Với $20$ số nguyên dương liên tiếp thì sẽ tồn tại $d$ có dạng $20k+15$, khi đó $d$ thỏa $(1)$ và $(2)$. Thật vậy

  • $d\equiv 3\pmod{4}$ nên $d$ không phải số chính phương
  • Tồn tại số nguyên tố $p\equiv 3\pmod{4}$ sao cho $p\mid 4k+3$, khi đó $p\mid n^2d$. Mặt khác $p\nmid a^2+1^2, a^2+2^2$ với mọi số nguyên $a$ nên $n^2d\neq m^2+1,m^2+4$
  • $5\mid n^2d$ nhưng $5\nmid a^2+2,a^2+3$ với mọi số nguyên $a$ nên $n^2d\neq m^2+2,m^2+3$

Với cách chọn $d$ như trên ta có được $n^2d-m\ge 5$ nên có được điều cần chứng minh.




#731721 Tìm $a,n$ nguyên dương để $a^{n^2+2n-1}-99$ là...

Đã gửi bởi nhungvienkimcuong on 22-11-2021 - 09:10 trong Số học

Tìm $a,n$ nguyên dương để $a^{n^2+2n-1}-99$ là số chính phương.

Giả sử $a^{n^2+2n-1}-99=b^2\ (1)$

$\bullet$ TH1: $n$ lẻ

Khi đó $n^2+2n-1=2m$. Ta viết lại $(1)$ dưới dạng $(a^m-b)(a^m+b)=99$. Tới đây tìm được

$$(a,n)\in \{(10,1),(18,1),(50,1)\}$$

$\bullet$ TH1: $n$ chẵn

Ta sẽ chứng minh không tồn tại nghiệm trong trường hợp này. Nhận thấy $n^2+2n-1=4k+3$

  • Nếu $a$ chẵn thì $b^2\equiv 5\pmod{8}$ (vô lí)
  • Nếu $a\equiv 1\pmod{4}$ thì $b^2\equiv 2\pmod{4}$ (vô lí)
  • Nếu $a\equiv 3\pmod{4}$, ta viết lại $(1)$ dưới dạng

$$a^{4k+3}+1=b^2+10^2$$

Vì $(a^{4k+3}+1)/(a+1)\equiv 3\pmod{4}$ nên tồn tại số nguyên tố $p\equiv 3\pmod{4}$ là ước của $a^{4k+3}+1$. Suy ra $p\mid b^2+10^2$, điều này dẫn tới $p\mid 10$ (vô lí)




#731842 Đề thi chọn đội tuyển lớp 12 - VMO - Bà Rịa-Vũng Tàu 2022

Đã gửi bởi nhungvienkimcuong on 30-11-2021 - 20:19 trong Thi HSG cấp Tỉnh, Thành phố. Olympic 30-4. Đề thi và kiểm tra đội tuyển các cấp.

Bài 1 (4,0 điểm).

b) Tìm tất cả bộ ba số thực dương $(x,y,z)$ thoả mãn hai điều kiện $xy+yz+zx+xyz=4$ và $\sqrt{2(4-xy)}+ \sqrt{5(4-yz)}+ \sqrt{10(4-zx)}=12$.

 

Với điều kiện $xy+yz+zx+xyz=4$ thì tồn tại các số dương $a,b,c$ sao cho (xem thêm ở đây)

$$x=\frac{2b}{a+c},\ y=\frac{2c}{a+b},\ z=\frac{2a}{b+c}$$

Thay vào điện kiện thứ hai ta có được

$$\sqrt{\frac{2a(a+b+c)}{(a+b)(a+c)}}+\sqrt{\frac{5b(a+b+c)}{(b+c)(b+a)}}+\sqrt{\frac{10c(a+b+c)}{(c+a)(c+b)}}=6 \tag{1}$$

Với số $6$ và vai trò các biến không đối xứng thì mình nghĩ đến việc tách $6=1+2+3$. Thử bộ số $(1,2,3)$ vào và nháp xíu thì tìm được $(a,b,c)\sim (1,2,3)$ (quá đẹp rồi  :D ). Ta có 

$$VT(1)\le \frac{1}{2}\left ( \frac{2a(a+b+c)}{(a+b)(a+c)}+1+\frac{1}{2}\left ( \frac{5b(a+b+c)}{(b+c)(b+a)}+4 \right )+\frac{1}{3}\left ( \frac{10c(a+b+c)}{(c+a)(c+b)}+9 \right ) \right )$$

Đến đây ta sẽ chứng minh 

$$\frac{1}{2}\left ( \frac{2a(a+b+c)}{(a+b)(a+c)}+1+\frac{1}{2}\left ( \frac{5b(a+b+c)}{(b+c)(b+a)}+4 \right )+\frac{1}{3}\left ( \frac{10c(a+b+c)}{(c+a)(c+b)}+9 \right ) \right )\le VP(1)=6$$

Quy đồng thì thu được bất đẳng thức trên tương đương với

$$(9ab+bc+4ca)(a+b+c)\ge 36abc$$

Bất đẳng thức này luôn đúng vì theo bất đẳng thức Cauchy thì

$$(9ab+bc+4ca)(a+b+c)\ge 6\sqrt[6]{(3ab)^3\cdot bc\cdot (2ca)^2}\cdot 6\sqrt[6]{a\cdot (b/2)^2\cdot (c/3)^3}=36abc$$

Dấu bằng xảy ra khi và chỉ khi $(a,b,c)\sim (1,2,3)$, dẫn tới $(x,y,z)=\left ( 1,2,\frac{2}{5} \right )$.




#732372 Chứng minh rằng số tam giác diện tích bằng $1$ có các đỉnh thuộc...

Đã gửi bởi nhungvienkimcuong on 07-01-2022 - 06:04 trong Tổ hợp và rời rạc

Ta thấy rằng $\frac{2}{3}(n^2-n)=\frac{4}{3}\binom{n}{2}$. Do vậy việc chứng minh là chỉ ra: với hai điểm bất kì thuộc $n$ điểm đã cho thì có nhiều nhất $4$ điểm để tạo thành tam giác có diện tích bằng $1$ (chứng minh điều này không khó, xem hình bên dưới). Còn việc chia cho $3$ thì cũng hiển nhiên vì mỗi tam giác như thế sẽ được đếm ba lần.

 

Capture.PNG

 

 




#732861 CMR $a^2+3ab+3b^2-1$ chia hết cho lập phương của một số nguyên lớn...

Đã gửi bởi nhungvienkimcuong on 05-03-2022 - 14:39 trong Số học

Cho a,b nguyên dương sao cho $a+b^3\vdots a^2+3ab+3b^2-1$

CMR $a^2+3ab+3b^2-1$ chia hết cho lập phương của một số nguyên lớn hơn 1.

Đặt $T=a^2+3ab+3b^2-1$, khi đó

\[T\mid a+b^3+a(a^2+3ab+3b^2-1)\]

hay $T\mid (a+b)^3\quad$ $(\ast)$. Giả sử ta có phân tích thành thừa số nguyên tố như sau

\[a+b=\prod_{i=1}^kp_i^{c_i}\ (c_i>0)\quad \text{và}\quad T=\prod_{i=1}^kp_i^{t_i}\ (t_i\ge 0).\]

Từ $(\ast)$ suy ra $t_i\le 3c_i$. Giả sử $T$ không chia hết cho số nguyên nào có dạng $c^3\ (c>1)$ thì $t_i\in \{0,1,2\}$ với mọi $i=\overline{1,k}$, do vậy ta có

\[t_i\le 2\le 2c_i,\ \forall i=\overline{1,k}\implies T\mid (a+b)^2.\]

Từ đây dễ thấy mâu thuẫn vì $(a+b)^2<T$.




#732951 $x^n - y^n \notin \mathbb{Z}$

Đã gửi bởi nhungvienkimcuong on 15-03-2022 - 21:11 trong Số học

Cho x, y là hai số hữu tỉ không nguyên phân biệt. CMR tồn tại n nguyên dương để $x^n - y^n \notin \mathbb{Z}$

Ta sẽ chứng minh: với hai số hữu tỉ $x,y$ thỏa mãn $x^n-y^n\in \mathbb{Z}$ thì $x,y$ đều là số nguyên.

Gọi $c$ là số nguyên dương nhỏ nhất sao cho $cx$ và $cy$ đều là các số nguyên, khi đó đặt $x=\frac{a}{c},y=\frac{b}{c}$. Điều kiện $x^n-y^n\in \mathbb{Z}$ suy ra

$$c^n\mid a^n-b^n,\quad \forall n\ge 1.$$

Giả sử $c>1$, gọi $p$ là ước nguyên tố của $c$, vì $c\mid a-b$ nên $p\mid a-b$. Ta có thể giả sử $p\nmid a,b$ (để phù hợp với việc chọn $c$). Ta sẽ chứng minh mâu thuẫn khi $p$ lẻ (trường hợp $p=2$ xử lí tương tự), áp dụng bổ đề nâng lũy thừa (LTE) ta có

$$n\le v_p(c^n)\le v_p(a^n-b^n)=v_p(a-b)+v_p(n)$$

Từ dây suy ra 

$$p^n\le (x-y)n\implies x-y\ge \frac{p^n}{n}$$

Cho $n\to \infty$ dẫn tới vô lí.




#733085 Có một CLB chứa ít nhất 64 học sinh nam và 64 học sinh nữ

Đã gửi bởi nhungvienkimcuong on 02-04-2022 - 09:05 trong Tổ hợp và rời rạc

Một trường học có 2020 học sinh nam và 2020 học sinh nữ. Trong trường, người ta có tổ chức một số câu lạc bộ, biết rằng mỗi học sinh tham gia không quá 16 câu lạc bộ và mỗi cặp hai học sinh nam và nữ bất kì thì cùng tham gia vào ít nhất 1 câu lạc bộ. Chứng minh rằng có một câu lạc bộ trong trường mà trong đó có ít nhất 64 học sinh nam và 64 học sinh nữ

Giả sử mỗi CLB có nhiều nhất 63 học sinh nam hoặc 63 học sinh nữ. Ta sẽ đếm $T$ là số bộ $(\{g,b\},c)$ hay số cặp học sinh khác giới cùng tham gia một CLB.

$\bullet$ Với mỗi cặp học sinh khác giới $\{g,b\}$, khi đó tồn tại ít nhất một CLB $c$. Do vậy

$$T\ge 2020^2\qquad (1)$$

$\bullet$ Ta chia các CLB ban đầu thành tập hợp $\mathcal{C}_b$ là tập hợp các CLB có nhiều nhất 63 học sinh nam và $\mathcal{C}_g$ là tập hợp các CLB có nhiều nhất 63 học sinh nữ.

Có $2020\cdot 16\cdot 63$ bộ $(\{g,b\},c)$ trong đó CLB $c\in \mathcal{C}_b$. Vì có $2020$ cách chọn học sinh nữ, mỗi học sinh nữ như vậy tham gia nhiều nhất $16$ CLB, mỗi CLB như thế có nhiều nhất $63$ học sinh nam.

Hoàn toàn tương tự thì có $2020\cdot 16\cdot 63$ bộ $(\{g,b\},c)$ với $c\in \mathcal{C}_g$. Do vậy

$$T\le 2020\cdot 16\cdot 63+2020\cdot 16\cdot 63\qquad (2)$$

Từ $(1)$ và $(2)$ suy ra

$$2020\cdot 16\cdot 63+2020\cdot 16\cdot 63\ge 2020^2\ (\text{vô lí})$$




#733191 Hàm trên tập số nguyên dương $ f(mn) = f(m)f(n)$ với mọi số nguyên...

Đã gửi bởi nhungvienkimcuong on 11-04-2022 - 14:31 trong Phương trình hàm

Tìm tất cả các hàm số $f: \mathbb{N}^{*}  \mapsto \mathbb{N}^{*}$ thỏa mãn đồng thời $3$ điều kiện:

 

$1/$ $ f(2) =2$

 

$2/$ $  f(mn) = f(m)f(n)$ với mọi số nguyên dương $m;n$

 

$3/$ $ f(m) < f(n)$ với mọi $m <n$

Một nhận xét quan trọng trong bài này chính là: nếu $f(k)=k$ thì $f(i)=i$ với mọi $i<k$.

Do vậy ta chỉ cần chứng minh tồn tại một dãy số nguyên $(a_n)$ có $\lim a_n=+\infty$ thỏa mãn $f(a_n)=a_n$ với mọi $n$. Có thể xây dựng dãy như này khá thoải mái, sau đây là một ví dụ.

$$(a_n):\left\{\begin{matrix}a_1=3,\\ a_{n+1}=a_n^2-a_n.\end{matrix}\right.$$

Chứng minh $f(3)=3$ thì xin nhường bạn đọc. Phần còn lại thì quy nạp đơn giản rồi, nếu $f(a_n)=a_n$ thì $f(a_n-1)=a_n-1$ (nhận xét). Dẫn đến

$$f(a_{n+1})=f(a_n(a_n-1))=f(a_n)f(a_n-1)=a_n(a_n-1)=a_{n+1}.$$

Ngoài ra dãy $(a_n)$ tăng nên $f(n)=n$ với mọi $n\in \mathbb{N}^*$.




#733316 $2(ab+bc+ca)-3abc\geqslant \sum a\sqrt{\frac...

Đã gửi bởi nhungvienkimcuong on 25-04-2022 - 20:02 trong Bất đẳng thức và cực trị

Bài này sử dụng bất đẳng thức phụ sau đây là ra

$$\sqrt{2(x^2+y^2)}=\frac{\sqrt{2(x^2+y^2)(x+y)^2}}{x+y}\le \frac{2(x^2+y^2)+(x+y)^2}{2(x+y)}=\frac{3}{2}(x+y)-\frac{2xy}{x+y}.$$




#733671 Xây dựng hàm $f$ thỏa bài toán và tính $f(0)$

Đã gửi bởi nhungvienkimcuong on 16-06-2022 - 10:48 trong Phương trình hàm

Kiểm tra được hàm $f(x)=\left\{\begin{matrix}0,\quad \text{với}\ x=0\qquad\\ 1,\quad\text{với}\ 0<x\le 1\end{matrix}\right.$ thỏa đề.

 

Ta sẽ chứng minh $f(0)=0$ bằng phản chứng, giả sử $f(0)\neq 0$. Thay $x=y=0$ ta có $f(f(0))=2f(0)$, với $x=0$ thì $$f(f(0)+y)=f(0)+f(y)\quad (\ast)$$

Vì $0<f(0)\le 1$ nên tồn tại số nguyên dương $n$ sao cho $\frac{1}{n+1}<f(0)\le \frac{1}{n}$, hay nói cách khác $nf(0)\le 1$ và $(n+1)f(0)>1$.

Lần lượt thay $y$ bằng $f(0),\ 2f(0),\ \dots,\ (n-1)f(0)$ vào $(\ast)$ ta có

$$f(2f(0))=3f(0),\quad f(3f(0))=4f(0),\quad \dots,\quad f(nf(0))=(n+1)f(0)$$

Đẳng thức $f(nf(0))=(n+1)f(0)$ dẫn tới điều mâu thuẫn.




#733679 Xây dựng hàm $f$ thỏa bài toán và tính $f(0)$

Đã gửi bởi nhungvienkimcuong on 17-06-2022 - 08:05 trong Phương trình hàm

Cái chỗ thay này không hợp lý nha, do với $n$ đủ lớn thì $(n-1)f(0) $ có thể lớn hơn $1$ , trong khi $y \in [0;1]$ nên đâu có thay vậy được.

Đó là em viết theo dòng suy nghĩ của bản thân nên chưa được phù hợp với giả thiết lắm, đảo câu tí là oke  :D




#733722 $f(ab) = f(a)+f(b) + k f ( \gcd(a,b))$

Đã gửi bởi nhungvienkimcuong on 20-06-2022 - 15:25 trong Phương trình hàm

Tìm tất cả các số nguyên dương $k$ sao cho tồn tại  hàm số $f: \mathbb{N}^{*}$ $\mapsto$ $\mathbb{Z}$ thỏa mãn đồng thời $2$ điều kiện:

Theo mấy cái định nghĩa của hàm số ở VN thì em toàn thấy là $f: \mathbb{N}^*\to \mathbb{Z},\ x\mapsto f(x)$. Thế nên $\mapsto$ mà anh hay dùng theo em là chưa hợp lí (có gì không đúng thì anh lượng thứ  :icon6: )

 

Quay lại bài toán thì yêu cầu chỉ cần tìm $k$ nguyên dương khá đơn giản, sẽ đổi lại là tìm số nguyên $k$. Thay $a=b=1$ ta có

$$f(1)=2f(1)+kf(1)\implies (k+1)f(1)=0$$

$\bullet$ Xét trường hợp $f(1)=0$, thay $b=1$ và $a$ bất kì vào giả thiết ta có

$$f(a)=f(a)+f(1)+kf(\gcd(a,1))\implies kf(a)=0$$

Vì $f(1997)\neq 0$ nên $k=0$. Với $k=0$, dễ thấy hàm số

$$f\left(p_1^{\alpha_1}\dots p_t^{\alpha_t} \right )=\alpha_1f(p_1)+\dots+\alpha_tf(p_t)$$

thỏa đề, trong đó $f(1997)=1998$ và $f(p)=0$ với mọi $p\neq 1997$.

$\bullet$ Trường hợp $k=-1$ thì hàm $f\equiv 1998$ thỏa đề.

Vậy $k\in \{-1,0\}$.




#733761 $f(2n-1) = 2^n ; f(2n) = n + \frac{2n}{d(n)}...

Đã gửi bởi nhungvienkimcuong on 22-06-2022 - 15:22 trong Phương trình hàm

Với mỗi số nguyên dương $n$, ta ký hiệu $d(n)$ là ước số lẻ lớn nhất của $n$

 

Hàm số $f: \mathbb{N}^{*} \to \mathbb{N}^{*}$ thỏa mãn:

 

$f(2n-1) = 2^n ; f(2n) = n + \frac{2n}{d(n)}$  với mọi $n \in \mathbb{N}^{*}$

 

Tìm tất cả các số nguyên dương $k$ sao cho : $ f^k (1) = f(f(...(f(1))...) = 1997$ ($k$ lần lặp $f$)

Dễ dàng tính được với số $y$ lẻ thì $f(2^xy)=2^{x-1}(y+2)$. Do vậy sau $x$ lần tác động $f$ vào số chẵn $2^xy$ thì ta mới thu được số lẻ (số lẻ đó là $y+2x$).

$$2^xy\overset{f}{\longrightarrow}2^{x-1}(y+2)\overset{f}{\longrightarrow}\cdots\overset{f}{\longrightarrow}y+2x$$

Từ nhận xét trên ta thu được: cần $m+1$ lần tác động $f$ vào số lẻ $2m-1$ thì ta mới thu được số lẻ tiếp theo (số lẻ đó là $2m+1$).

$$2m-1\overset{f}{\longrightarrow}2^m\overset{f}{\longrightarrow}\cdots\overset{f}{\longrightarrow}2m+1$$

Với nhận xét này ta thấy rằng khi bắt đầu bởi số lẻ $1$ thì sẽ tồn tại duy nhất $k$ sao cho $f^k(1)=1997$ (vì các số lẻ thu được sau các lần tác động $f$ tăng dần). 

$$1\overset{2}{\longrightarrow}3\overset{3}{\longrightarrow}\cdots\overset{999}{\longrightarrow}1997$$

Vậy $k=2+3+\dots+999=\frac{999\cdot 1000}{2}-1=499499$.




#733786 $$x^{p-1}+x^{p-2}+...x+2=y^{n+1}...

Đã gửi bởi nhungvienkimcuong on 25-06-2022 - 07:12 trong Số học

Bài toán. (Sưu tầm) Cho $p$ là một số nguyên tố lớn hơn $3$, $n$ là số nguyên dương sao cho các số $p-1,p,n,n+1$ từng đôi một không có ước số chung lớn hơn $2$.Chứng minh rằng phương trình sau không có nghiệm nguyên dương $x,y$

$$x^{p-1}+x^{p-2}+...x+2=y^{n+1}$$

Bài này tương tự ở đây. Trường hợp 2 thì xử lí như sau: Với $y-1\equiv 1\pmod{p}$ thì

$$y^n+y^{n-1}+\dots+1\equiv 2^n+2^{n-1}+\dots+1=2^{n+1}-1\pmod{p}.$$

Tới đây ta thấy rằng $\text{ord}_p(2)\mid n+1$, mặt khác $\text{ord}_p(2)\mid p-1$ nên

$$\text{ord}_p(2)\mid \gcd(n+1,p-1)\implies \text{ord}_p(2)\le \gcd(n+1,p-1)\le 2 .$$

Khi đó $p\le 2^{\text{ord}_p(2)}-1\le 2^2-1=3$ (mâu thuẫn với giả thiết $p>3$). Vậy không tồn tại $x,y$ thỏa đề.




#733856 $0< | a+b \sqrt{2} + c \sqrt{3}| <...

Đã gửi bởi nhungvienkimcuong on 02-07-2022 - 07:21 trong Các dạng toán khác

Chứng minh rằng với mọi $\epsilon >0$ Tồn tại các số nguyên $a;b;c$ sao cho :

 

$0< | a+b \sqrt{2} + c \sqrt{3}| < \epsilon$

Ta thấy rằng tồn tại vô hạn $n$ nguyên dương thỏa mãn $2n^2-\left \lfloor n\sqrt{2} \right \rfloor^2=1$ (xem thêm ở đây). Khi đó

$$n\sqrt{2}-\left \lfloor n\sqrt{2} \right \rfloor=\frac{1}{n\sqrt{2}+\left \lfloor n\sqrt{2} \right \rfloor}<\frac{1}{2n\sqrt{2}-1}.$$

Vì tồn tại vô hạn $n$ nên hoàn toàn có thể chọn $n$ sao cho $\frac{1}{2n\sqrt{2}-1}<\epsilon$, dẫn tới 

$$0<\left|\left \lfloor n\sqrt{2} \right \rfloor-n\sqrt{2}+0\cdot\sqrt{3}\right|<\epsilon.$$

Vậy với $a=\left \lfloor n\sqrt{2} \right \rfloor, b=-n$ và $c=0$ thì thỏa đề.




#733869 $0< | a+b \sqrt{2} + c \sqrt{3}| <...

Đã gửi bởi nhungvienkimcuong on 03-07-2022 - 06:47 trong Các dạng toán khác

Nếu mình hiểu đúng thì mệnh đề này sẽ suy ra là tập $\{a+b\sqrt{q}\}$ có tính trù mật nhỉ?

Phải là ngược lại mới đúng á anh  :lol: . Vì tập $\left\{a+b\sqrt{q}:a,b\in \mathbb{Z}\right\}$ trù mật trong $\mathbb{R}$ nếu với hai số dương $\epsilon_1,\epsilon_2$ bất kì ($\epsilon_1<\epsilon_2$) thì tồn tại $a,b$ sao cho $\epsilon_1<\left|a+b\sqrt{q}\right|<\epsilon_2$.




#733874 Trại hè hùng vương 2019

Đã gửi bởi nhungvienkimcuong on 03-07-2022 - 10:37 trong Thi HSG cấp Tỉnh, Thành phố. Olympic 30-4. Đề thi và kiểm tra đội tuyển các cấp.

Bài 5: (Đếm bằng hai cách cho "cặp cạnh cùng màu")

Ở đây ta quan tâm đến các cặp cạnh cùng màu và cùng xuất phát từ một điểm, ta sẽ gọi các cặp cạnh như vậy là "cặp cạnh đẹp". Dễ thấy tam giác có ba cạnh cùng màu thì sẽ có 3 cặp cạnh đẹp, và tam gíac có ba cạnh không cùng màu thì có 1 cặp cạnh đẹp.

Capture.PNG

Gọi $x$ là số tam giác có ba cạnh cùng màu, khi đó còn lại $20-x$ tam giác có ba cạnh không cùng màu. Dẫn đến
\[\#\{\text{cặp cạnh đẹp}\}=3x+(20-x)=2x+20.\tag{1}\]
Tại điểm $A$, gọi $a$ là số cạnh màu đỏ có $A$ là đầu mút, do đó có $5-a$ số cạnh màu xanh có $A$ là đầu mút. Số cặp cạnh đẹp thu được từ điểm $A$ là
\[\binom{a}{2}+\binom{5-a}{2}\ge \binom{2}{2}+\binom{3}{2}= 4.\]
Do vậy
\[\#\{\text{cặp cạnh đẹp}\}\ge \sum_{\text{điểm}\ A}4=24.\tag{2}\]
Từ $(1)$ và $(2)$ thu được $x\ge 2$, đây cũng chính là điều cần chứng minh.

 

Ghi chú: Sử dụng "cặp cạnh cùng màu" đã được viết trong một tập san mà Diễn đàn biên soạn năm 2006 (tải về tại đây, mục 8 - góc cùng màu).




#733880 $(P(x))^4+2P(x)+2=(x^4+2x^2+2)G(x)+3H(x)$

Đã gửi bởi nhungvienkimcuong on 03-07-2022 - 15:05 trong Đa thức

Tìm đa thức $P\in \mathbb{Z}[x]$ có bậc nhỏ nhất sao cho tồn tại hai đa thức $G,H\in \mathbb{Z}[x]$ thỏa mãn

$$(P(x))^4+2P(x)+2=(x^4+2x^2+2)G(x)+3H(x),\quad \forall x\in \mathbb{R}.$$

(Trường đông Hàn Quốc 2021)

Bài này mình làm cơ bắp trên $\mathbb{Z}_3[x]$, phải sử dụng kiến thức về trường (toán cao cấp). Chắc hẳn phải có một lời giải đẹp và phổ thông hơn, mong nhận được sự góp ý từ mọi người  :icon6:




#734038 $3x^2+2y^2-6=xyk$

Đã gửi bởi nhungvienkimcuong on 19-07-2022 - 15:15 trong Số học

Về mặt ý tưởng, với hình thức đề bài như này thì liên tưởng ngay tới phương trình Pell, với yêu cầu chứng minh thì ta thấy ngay câu chuyện xây dựng dãy nghiệm, do vậy liên tưởng ban đầu là hoàn toàn hợp lí. Giờ thì xử lí bài toán thôi  :D

Biết rằng các số nguyên dương $x,y$ thỏa mãn $4\nmid x,3\nmid y$ và

$$3x^2+2y^2-6=kxy$$

trong đó số nguyên $k\ge 8$. Chứng minh rằng tồn tại vô hạn cặp số $(x,y)$ như thế.

 

Phương trình $3x^2+2y^2-6=kxy$ tương đương $(4y-kx)^2-(k^2-24)x^2=48$. Do vậy ta sẽ chứng minh: Biết rằng $A,B,k$ là các số nguyên dương $(k\ge 8)$ thỏa mãn phương trình

$$A^2-(k^2-24)B^2=48\qquad (\blacklozenge)$$

trong đó $4\nmid B$ và $\frac{A+kB}{4}$ là số nguyên, ngoài ra thì $3\nmid \frac{A+kB}{4}$. Khi đó tồn tại vô hạn cặp số $(A,B)$ cũng thỏa mãn các điều kiện vừa nêu.

(Ở đây đang làm cho trường hợp $4y\ge  kx$ nên mới xét $A+kB$, nếu $4y< kx$ xử lí tương tự cho $kB-A$ nhưng mất thời gian hơn xíu)

 

Bước 1: Xây dựng dãy nghiệm cho phương trình $(\blacklozenge)$.

 

Bước 2: Chứng minh $\frac{A_n+kB_n}{4}\in \mathbb{Z}$ và $3\nmid \frac{A_n+kB_n}{4}$ với mọi $n$.

 

Bước 3: Chứng minh $4\nmid B_n$ với mọi $n$ lẻ.




#734050 $\int f(\lfloor x\rfloor)dx…$

Đã gửi bởi nhungvienkimcuong on 21-07-2022 - 08:35 trong Tích phân - Nguyên hàm

Chứng minh rằng:
$$\boxed{\int_1^x \bigg(-\frac{\lfloor t\rfloor}{t}+\sum_{k=1}^{\lfloor t\rfloor}\frac{1}{k}\bigg)dt=-\lfloor x\rfloor(1+\ln x)+\ln(\lfloor x\rfloor !)+x\sum_{k=1}^{\lfloor x\rfloor} \frac{1}{k},\,\,\,\,\,\,\,(x\ge 1)}$$

 

Kí hiệu tích phân vế trái là $I$ và chuỗi điều hòa $H(n):=1+\frac{1}{2}+\dots+\frac{1}{n}$. Biến đổi

\begin{align*}I&=\sum_{m=1}^{\left \lfloor x \right \rfloor-1}\int_{m}^{m+1}\left ( \frac{-\left \lfloor t \right \rfloor}{t}+H( \left \lfloor t \right \rfloor) \right )\mathrm{d}t+\int_{\left \lfloor x \right \rfloor}^{x}\left ( \frac{-\left \lfloor t \right \rfloor}{t}+H( \left \lfloor t \right \rfloor) \right )\mathrm{d}t \\ &=\sum_{m=1}^{\left \lfloor x \right \rfloor-1}\underbrace{\int_{m}^{m+1}\left ( \frac{-m}{t}+H( m) \right )\mathrm{d}t}_{A(m)}+\underbrace{\int_{\left \lfloor x \right \rfloor}^{x}\left ( \frac{-\left \lfloor x \right \rfloor}{t}+H( \left \lfloor x \right \rfloor) \right )\mathrm{d}t}_{B}.\qquad (\blacklozenge)\end{align*}

Dễ dàng tính được

\begin{align*}A(m)=m(\ln m-\ln(m+1))+H(m)\qquad \text{và}\qquad B=-\left \lfloor x \right \rfloor(\ln x-\ln \left \lfloor x \right \rfloor)+(x-\left \lfloor x \right \rfloor)H(\left \lfloor x \right \rfloor).\end{align*}

Thay vào $(\blacklozenge)$ và rút gọn thu được

\begin{align*} I&=\sum_{m=1}^{\left \lfloor x \right \rfloor-1}A(m)+B\\&=\ln\left ( \left \lfloor x \right \rfloor! \right )-\left \lfloor x \right \rfloor\ln x+\sum_{m=1}^{\left \lfloor x \right \rfloor-1}H(m)+(x-\left \lfloor x \right \rfloor)H(\left \lfloor x \right \rfloor).\end{align*}

Phần chưa liên quan đến biểu thức cần chứng minh là tổng của các chuỗi điều hòa, dễ thấy

$$\sum_{m=1}^{\left \lfloor x \right \rfloor-1}H(m)=\sum_{m=1}^{\left \lfloor x \right \rfloor-1}\frac{\left \lfloor x \right \rfloor-m}{m}=\left \lfloor x \right \rfloor H(\left \lfloor x \right \rfloor-1)-\left \lfloor x \right \rfloor+1.$$

Thay vào trên thu được điều cần chứng minh.




#734052 $f:\mathbb{Z}\to \mathbb{Z}$ tho...

Đã gửi bởi nhungvienkimcuong on 21-07-2022 - 11:14 trong Đa thức

Cho $f:\mathbb{Z}\to \mathbb{Z}$ thoả mãn $d\mapsto f(d)-f(d-1)$ là một hàm đa thức. Chứng minh rằng $f$ cũng là một hàm đa thức.

Bài này em không hiểu ý đề bài  :wacko:.  Hàm $f(x)=\left \lfloor x \right \rfloor$ thỏa đề nhưng đâu phải hàm đa thức?




#734065 $m,n,N,k \in \mathbb{N}$ thỏa mãn $(n^...

Đã gửi bởi nhungvienkimcuong on 22-07-2022 - 07:10 trong Số học

Bài này nhìn đồ sộ nhưng thật ra chỉ cần xét tính chẵn lẻ của $n$ và làm việc trên $\text{mod}\ 4$ là ra.

Ngoài ra thì bài này từng xuất hiện trong cuộc thi Marathon cấp THCS năm 2014 của diễn đàn (Xem ở đây)